9
$\begingroup$

The "classical Beurling density" of a subset of the natural numbers is $d(A)=lim_{n\rightarrow\infty}\frac{|A\cap[1,n]|}{n}$, when it exists. It defines a finitely additive probability measure on the natural numbers which is invariant with respect to the sum. Here is my question: does there exist a "nice formula" to describe a finitely additive probability measure on $\mathbb N$ which is invariant with respect to the multiplication?

A couple of remarks: I don't know if it is trivial that such a measure exists, but anyway it follows from the application of a general result of Vern Paulsen (on arxiv "Syndetic sets and amenability"). Another problem would be that of finding the measure of particular sets. What about the measure of {$1!,2!,3!,4!...$}? Sets with measure differente from 0 and 1? For example the set of numbers whose first digit through 4 to 9 seems to have measure $=\log_{10}4$... any other?

Thanks in advance, Valerio

$\endgroup$
9
  • $\begingroup$ when you say invariant with respect to the sum do you mean that $\lbrace k+2 \mid k \in A \rbrace$ has the same measure as $A$? $\endgroup$ Mar 21, 2011 at 21:52
  • $\begingroup$ Any finitely additive probability measure (also called a content) is defined on a field. But the collection of subsets $A$ of the natural numbers having a density $d(A)$ is not a field. $\endgroup$
    – Did
    Mar 21, 2011 at 22:37
  • $\begingroup$ That is, one can write down two sets that have density, but whose intersection does not have density. $\endgroup$ Mar 21, 2011 at 23:48
  • $\begingroup$ One example would be a measure assigning $\mu(A)=1$ if $A$ contains $0$, and $\mu(A)=0$ otherwise. $\endgroup$ Mar 22, 2011 at 0:59
  • $\begingroup$ @Daniel, for many of us, $0$ is not a natural number. $\endgroup$ Mar 22, 2011 at 4:31

3 Answers 3

7
$\begingroup$

The natural thing to do here is to replace the intervals $[1,n]$ (and $n = |[1,n]|$) in the definition of $d(A)$ with a sequence $F_n$ of subsets of $\mathbb{N}$ which is multiplicatively asymptotically invariant (or, in other words, a Folner sequence for the semigroup $(\mathbb{N},\cdot)$). For an exploration of this idea, as well as applications, see for instance this article by Vitaly Bergelson:

Multiplicatively large sets and ergodic Ramsey theory, Israel Journal of Mathematics 148 (2005), 23-40.

EDIT: One particular example (mentioned in the article) is to take $F_n$ to be the set of all positive integers which can be written as a product of powers of the first $n$ primes, where the powers are allowed to be any non-negative integer which is less than or equal to $n$. The motivation for choosing $F_n$ this way is that just as $1$ generates the additive semigroup $(\mathbb{N},+)$, the primes generate the multiplicative semigroup. Think of balls in the corresponding Cayley graphs with radius getting larger and larger. The article contains many other examples of such $F_n$, and each of them gives a notion of "multiplicative density" by setting $d(A) = \lim_{n \to \infty} \frac{|A \cap F_n|}{|F_n|}$ (if the limit exists. Otherwise one usually considers the limsup and liminf).

$\endgroup$
6
  • $\begingroup$ Perhaps I am misunderstanding you, but I don't think this works; this is what Gerald Edgar attempts in his answer. It seems to me that $\mathbb{N}$ still has density $1$, whereas $2\mathbb{N}$ has density $1/2$. $\endgroup$ Mar 22, 2011 at 19:48
  • $\begingroup$ No, $A_k = [1,k!]$ is not a Folner sequence for $(\mathbb{N},\cdot)$, since $2[1,k!]=[2,2 \cdot k!]$ and $[1,k!] \triangle [2,2 \cdot k!]$ has cardinality $1+ k!$, which is not $o(|A_k| = k!)$. $\endgroup$
    – Mark
    Mar 22, 2011 at 21:01
  • $\begingroup$ Ah, I see. (Btw, $2[1, k!]$ is only the evens in $[2, 2k!]$, but your computations still works). What is an example of a Folner sequence for $\mathbb{N}$? $\endgroup$ Mar 22, 2011 at 22:09
  • $\begingroup$ Right. The notation confused me with intervals in the real numbers. As for your question, I've edited my answer to include such an example. $\endgroup$
    – Mark
    Mar 22, 2011 at 23:09
  • $\begingroup$ Thanks a lot, Mark. Even if I can't open the article, from Bergelson's home page directly either.. don't understand why! $\endgroup$ Mar 25, 2011 at 8:10
4
$\begingroup$

Here is an example. Let $d(A)$ be as you define it in the question, namely $$d(A)=\lim_{n\to \infty} \frac{|A\cap [1,n]|}{n}.$$ Let $U_n=\{k\in\mathbb{N} \text{ such that } k \text{ is a mutiple of } n!\}$. Then define $$\mu(A)=\lim_{n\to \infty} n!\cdot d(A\cap U_n).$$ Then I claim that $\mu$ is finitely additive and multiplicatively invariant. Finite additivity is obvious. For multiplicative invariance, note that for $s>k$, we have $(s+k)!\cdot d(kA\cap U_{s+k})=s!\cdot d(A\cap U_{s})$, unless I've screwed something up.

EDIT: Note by the way that one can replace the limit in the definition with $d$ with the Cesaro mean, for example, giving a much broader class of sets with defined measure. For example, with this addition, the set of natural numbers with a fixed leading digit in a fixed prime base $p$ has density $1/(p-1)$.

$\endgroup$
4
  • $\begingroup$ What would be an example of a set with density strictly between zero and one? $\endgroup$ Mar 22, 2011 at 6:08
  • $\begingroup$ It seems to me that numbers whose first digit is $1$ in base $3$ works, for example. $\endgroup$ Mar 22, 2011 at 6:14
  • $\begingroup$ (That is, leading, not trailing.) $\endgroup$ Mar 22, 2011 at 6:15
  • 1
    $\begingroup$ Another, more interesting example, seems to be--natural numbers $n$ such that the exponent of the greatest power of $2$ dividing $n$ equals twice that of the greatest power of $3$ dividing $n$, say $\pm 2$. $\endgroup$ Mar 22, 2011 at 8:03
0
$\begingroup$

How about: $$ \lim_{k\to\infty} \frac{|A \cap [1,k!]|}{k!} $$ when it exists?

$\endgroup$
7
  • $\begingroup$ Then all the positive integers have density 1, but the even integers still have density 1/2. Right? $\endgroup$
    – JBL
    Mar 22, 2011 at 0:13
  • $\begingroup$ (In fact, your proposed definition must agree with the usual one whenever the usual one exists.) $\endgroup$
    – JBL
    Mar 22, 2011 at 0:14
  • $\begingroup$ I would argue that the two measures disagree. Take the set of integers $A:=\cup_{n\in\mathbb{N}}[n!,\mathrm{E}(\sqrt{n}n!)]$. If I'm not mistaken, you should have that the sequence $\frac{A\cap[1,k]}{k}$ has both 0 and 1 as adherent values. $\endgroup$ Mar 22, 2011 at 0:36
  • $\begingroup$ while $\frac{|A\cap [1,k!]|}{k!}$ tends to $0$. $\endgroup$ Mar 22, 2011 at 0:38
  • $\begingroup$ Which is consistent with what I wrote: Gerald Edgar's density is the limit of a subsequence of the sequence whose limit is the usual density. $\endgroup$
    – JBL
    Mar 22, 2011 at 0:48

Your Answer

By clicking “Post Your Answer”, you agree to our terms of service and acknowledge you have read our privacy policy.

Not the answer you're looking for? Browse other questions tagged or ask your own question.